Linear: Find a set of basic solutions and show as linear combination

Click For Summary
The discussion revolves around finding a set of basic solutions for a system of linear equations and expressing the general solution as a linear combination of these solutions. The equations are reduced to a simpler form, revealing free variables b and e. Participants emphasize that any values can be assigned to b and e, leading to a variety of solutions. The final expression for the general solution can be constructed by filling in the placeholders with the appropriate coefficients derived from the equations. Ultimately, every solution can be obtained by varying the values of b and e, confirming the generality of the solution.
sumtingwong59
Messages
4
Reaction score
0

Homework Statement


Find a set of basic solutions and express the general solution as a linear combination of these basic solutions

a + 2b - c + 2d + e = 0
a + 2b + 2c + e = 0
2a + 4b - 2c + 3d + e = 0

Homework Equations


3. The Attempt at a Solution [/B]
i reduced it to:
1 2 0 0 -1 0
0 0 1 0 2/3 0
0 0 0 1 1 0

a = -2s + t
c = -2/3t
d = -t

I'm just not sure how i find solutions now. It could be literally anything could it not?
 
Physics news on Phys.org
Assuming your arithmetic is correct (I didn't check), you have let the free variables ##b = s## and ##e = t##. I'm going to leave them as ##b## and ##e##. Write your solution as$$
\left (\begin{array}{c}
a\\b\\c\\d\\e
\end{array}\right)
=
\left (\begin{array}{c}
-2b+e\\b\\-\frac 2 3 e\\-e\\e
\end{array}\right)
=
b\left (\begin{array}{c}
?\\?\\?\\?\\?
\end{array}\right)
+ e
\left (\begin{array}{c}
?\\?\\?\\?\\?
\end{array}\right)
$$Fill in the ?'s and you will have it.
 
Last edited:
LCKurtz said:
Assuming your arithmetic is correct (I didn't check), you have let the free variables ##b = s## and ##e = t##. I'm going to leave them as ##b## and ##e##. Write your solution as$$
\left (\begin{array}{c}
a\\b\\c\\d\\e
\end{array}\right)
=
\left (\begin{array}{c}
-2b+e\\b\\-\frac 2 3 e\\-e\\e
\end{array}\right)
=
b\left (\begin{array}{c}
?\\?\\?\\?\\?
\end{array}\right)
+ c
\left (\begin{array}{c}
?\\?\\?\\?\\?
\end{array}\right)
$$Fill in the ?'s and you will have it.

Do I just pick any number to plug into b and e, and then pick different numbers and plug them into the c bracket?
 
Do I just want to make it so each equation equals 0?
 
sumtingwong59 said:
Do I just pick any number to plug into b and e, and then pick different numbers and plug them into the c bracket?

The ##c## in front of the last bracket was a typo; I have corrected it to ##e##. Fill in the ?'s and there is nothing left to do. Every solution can be gotten for some value of ##b## and ##e##. That is the general solution.
 
Last edited:
Question: A clock's minute hand has length 4 and its hour hand has length 3. What is the distance between the tips at the moment when it is increasing most rapidly?(Putnam Exam Question) Answer: Making assumption that both the hands moves at constant angular velocities, the answer is ## \sqrt{7} .## But don't you think this assumption is somewhat doubtful and wrong?

Similar threads

Replies
12
Views
2K
  • · Replies 2 ·
Replies
2
Views
2K
Replies
1
Views
2K
Replies
2
Views
1K
  • · Replies 5 ·
Replies
5
Views
1K
Replies
4
Views
2K
  • · Replies 1 ·
Replies
1
Views
2K
  • · Replies 10 ·
Replies
10
Views
2K
  • · Replies 10 ·
Replies
10
Views
2K